Théorème de Fermat - Page 2 — Les-mathematiques.net The most powerful custom community solution in the world

Théorème de Fermat

24

Réponses

  • Modifié (May 2022)
    En fait il change la définition de l'ensemble des entiers et il le définit comme étant $D_1=\{ \sqrt[3]{n}\mid n \in \mathbb{N} \setminus \{0;1;2\}\}$
    Voilà, voilà, donc dans $D_1$, son ensemble des entiers à lui, il y a des solutions pour l'équation $x^3+y^3=z^3$ qui ne vérifient pas $xyz=0$ c'est bel et bien n'importe quoi mais je vous avais dit que j'étais peu confiante sur les progrès de la science dans ce fil.
    In mémoriam de tous les professeurs assassinés dans l'exercice de leurs fonctions en 2023, n'oublions jamais les noms de Agnes-Lassalle et Dominique-Bernard qui n'ont pas donné lieu aux mêmes réactions sur ce forum (et merci à GaBuZoMeu)
  • Modifié (May 2022)
    Bon, alors reprenons calmement :
    Z =  (1/3) . [racine cubique de (x +y) .( x^2 -x y +y^2 ) ]
    pourquoi 1/3 devant ? Effectivement cette identité remarquable est remarquablement fausse, comme le verrait un lycéen un minimum réveillé, et en plus je ne vois pas le rapport avec ce qui suit.
    on définit D1 : l'ensemble des entiers  = ENT (  de toutes racine cubique cubique de n / n  appartient  à N \{0,1,2} 
    si j'ai bien compris ce qu'est D1, en fait D1 c'est $\N^*$ (petit exercice de L1).
     X^3  + Y^3 += Z^3     est  vraie dans  D1    pour x=10 , y= 8 
    l'affirmation n'est absolument pas démontrée, en plus la démonstration de Fermat lorsque $n=3$ n'est pas, de mémoire, très difficile et fait appel à des outils d'algèbre de L3, donc dire que l'on a un contre-exemple à Fermat dans $\N^*$ c'est complètement absurde.
    surtout  quand on sait  que D1 est dénombrable et qui a le méme cardinal que N  , ce qui contredit la démarche de  Wiles, dont l'inverstigation est parti à partir  de la théorie des Ensembles .pour aboutir à son théoréme  ..., ?
    Je ne comprends pas la dernière phrase. Tu remets en cause le cadre de la théorie des ensembles ? Certes avec une autre axiomatique, le résultat de Fermat pourrait devenir inexact. Mais là le théorème est que dans cette axiomatique, le théorème est vrai, et il a été prouvé (enfin, je fais confiance à la communauté, je n'ai pas le niveau pour comprendre la preuve et en plus ce n'est pas un sujet qui m'intéresse).
  • Il a un contre-exemple à Fermat dans D1 et comme il appelle D1 l'ensemble des entiers, affaire réglé pour lui, faut pas aller chercher plus loin à mon avis.
    Sauf que D1 contient des réels puisqu'il contient toutes les racines cubiques.
    In mémoriam de tous les professeurs assassinés dans l'exercice de leurs fonctions en 2023, n'oublions jamais les noms de Agnes-Lassalle et Dominique-Bernard qui n'ont pas donné lieu aux mêmes réactions sur ce forum (et merci à GaBuZoMeu)
  • Modifié (May 2022)
    Effacé  .
    Le 😄 Farceur


  • Moi j'avais interprété ENT comme une partie entière ...
  • Je ne suis pas vraiment sûre qu'il prenne vraiment la partie entière même s'il a écrit ENT mais dans les 2 cas, cela reste du n'importe quoi.
    In mémoriam de tous les professeurs assassinés dans l'exercice de leurs fonctions en 2023, n'oublions jamais les noms de Agnes-Lassalle et Dominique-Bernard qui n'ont pas donné lieu aux mêmes réactions sur ce forum (et merci à GaBuZoMeu)
  • Je crois que math2 a raison sur ENT pour dire partie entière 
    Le 😄 Farceur


  • Modifié (May 2022)
    Si j'ai bien compris la définition de $D_1$, en notant $u_n=n^{1/3}$, la suite est strictement croissante, $u_3\approx 1,4$ puis ensuite :
    $0\leq u_{n+1}-u_n \leq \frac13 n^{-2/3} \leq \frac13 3^{-2/3} \approx 0,16$
    ce qui permet de dire de $D_1=\N^*$ ; et même, en gros ceci dit qu'à partir de l'entier $2$, chaque entier est réalisé au moins six fois comme partie entière d'un élément de la suite $(u_n)_{n\geq 2}$.
  • Stp @Vassillia ne le déstabilise plus en disant que ce qu'il dit est du n'importe  quoi. Soyons gentils avec lui. Pas d'humiliations 
    Le 😄 Farceur


  • D1  est un sous ensemble de D /  toute Premiere décimale de R est nulle   , R =   racine cubique de ( X^3 + Y^3 )  , x & y appartenant à N \{0 , 1} 

    -racine cubique de  (2^3  + 2^3) =2.00...
    -racine cubique de  (4^3  + 3^3) =3.08...
    -racine cubique de  (5^3  + 8^3) =5.02...
    -racine cubique de  (6^3  + 4^3) =4.09...
    -racine cubique de  (7^3  + 10^3) =6.05...
    .
    .
    .
    -racine cubique de  (X^3  + Y^3) = a , 0 ,d2 ,d3 ,d4 , ....

    tous appartiennent à D1   >:)
  • Là je renonce à lire, mieux vaut ne pas poster cela sur arxiv effectivement
  • @math2 Bonjour.
    L 'auteur du fil est @gpascal. Je crois que ce @KAWKAW a en profité pour s'incruster dans la conversation. 
    Le 😄 Farceur


  • Ah peut-être, je croyais innocemment que KAWKAW était le collègue dont parlait gpascal. Mais si tu as raison, tu me rassures.
  • Modifié (May 2022)
    Je crois avoir compris que l'ensemble $D_1$ est l'ensemble des réels probablement positifs et dont la première décimale est nulle, autrement dit $D_1=\{x\in\R^+\mid \lfloor 10x \rfloor \in10\N\}$.
    Dans cet ensemble, il y a effectivement des solutions, par exemple, le triplet $(10, 7, (1343)^{\frac{1}{3}})$ est solution.
    Le reste est farfelu, néanmoins.
  • Modifié (May 2022)
    Ah, pour ma part je n'ai jamais pensé que Kawkaw pouvait être le collègue de Gpascal, il n'aurait pas fait cette erreur énorme, ou Gpascal n'aurait pas pensé qu'on pouvait en parler. C'est pourquoi j'écrivais "En plus, un impoli, qui vient dans une discussion pour parler de ses lubies."
    J'espère que Gpascal n'est pas trop choqué que vous ayez pu penser ainsi.
    Cordialement.
  • Il serait sans doute de bon ton que la modération scinde le fil en deux, du coup.
  • Il vaut mieux fermer ce fil.
    Si @gpascal veut présenter une preuve, il n'a qu'a ouvrir  un nouveau fil.
    Le 😄 Farceur


  • Désolé d'avoir pu penser cela ... L'absence de réponse de gpascal suite à ces interventions m'a induit bêtement en erreur. Effectivement, mieux vaut que gpascal ouvre un nouveau fil s'il le juge utile, ne serait-ce que pour nous faire savoir où cela sera publié.
  • LEGLEG
    Modifié (May 2022)
    Bonjour
     
    Je ne pense pas que cela soit utile que gpascal ouvre un autre fil , pour nous faire part de son identité remarquable ; sous prétexte qu'un intervenant croit avoir trouvé son identité remarquable (qui plus est, est fausse).

    Mais sauf erreur, quel rapport avec le théorème de Fermat ?
     
    Car il s"agit bien de trouver un triplet pythagoricien tel que : mis au carré on obtienne le théorème de Pythagore ... non ?

    Une racine cubique n'est pas un carré , pour avoir un triplet de cubes il faudrait avoir trois racines carrés de cubes dans le triplet pythagoricien et non pas trois racines cubiques ...

    Ou si on veut, il faudrait trois racines carrées de cubes ou autres puissances premières > 3 , qui mesurent les côtés d'un triangle rectangle . 

    Pour cela on a la définition et la preuve qui permet d'utiliser la formule de paramétrage , qui donne les côtés d'un triangle rectangle en entiers naturels non nul . Soit avec deux entiers $u$ et $v$ premiers entre eux et de parités différentes, ou encore avec deux entiers impairs $p$ et $q$  premiers entre eux.

    Il faudrait aussi, que le couple de paramètres $(u , v )$ ou $(p , q)$ soit choisis dans les cubes ou leurs racines carrées, en supposant au minimum , qu'il existe une solution... Comme c'est la cas pour $n = 2$. 

    D'où, si on veut que des entiers naturels non nuls dans le triplet pythagoricien il faut résoudre le cas des puissances paires $2n$ , avec $n$ premier $ > 2$ puisque le cas $2n = 4$ a été résolu par Fermat et aussi probablement, pour ces puissances paires $> 4$ avec sa méthode de descente infinie...

    Or on sait qu'il n'existe pas de solutions, de façon élémentaire , ni même de racines carrées de cubes ou d'une puissance première $> 3$; car en mettant le couple de paramètres au carré , on aurait deux solutions immédiates : par addition et soustraction avec le même couple de paramètres, ce qui est absurde.
     
  • Il existe  des entiers positifs  x, y,z  /  z= RC (x^3 + y^3 )  :wink:
    -  Il existe une et une seul RC déterminée et distncte dans D1 
    -  D1  est  dénombrable  de méme cardinal  que N 
    -  IL suffit d'une premiere décimal de RC soit nulle pour que x,y,z  entiers positifs existent  !    


    * RC  :  racine cubique
  • Modifié (May 2022)
    Bravo kawkaw, tu publies. Tu deviendras célèbre.
    Le 😄 Farceur


  • Désolé, gebrane

    C'est moi qui me suis jeté sur la fabuleuse trouvaille de kawkaw. J'ai immédiatement envoyé à Arxiv son résultat que j'ai dit mien. Dix minutes à peine et me voilà publié (j'ai le mail de Arxiv qui atteste que c'est moi qui ai trouvé). On croit souvent que Arxiv c'est lent, qu'avant de te répondre, ils chipotent pour voir si une pouille ne traîne pas dans le cottage. "Là, c'est du Ramanujan" qu'ils m'ont dit illico. Ils sont prêts à me payer une somme exorbitante si je les autorise à publier "où ils veulent".
    Alors là, j'ai dit: NON!, des fois que kawkaw pige que je lui ai mis à l'envers.
    Bon, je n'ai pas été très correct. Rongé par la culpabilité, tenant à l'estime des estimables de ce forum, j'attends d'eux qu'ils me conseillent sur la suite à donner à cette affaire douteuse. C'est vrai qu'un million d'euros, même pour moi, ce n'est pas négligeable.
  • Je te conseille de prendre l'argent sans scrupules. KAWKAW devra se satisfaire des cacahuètes... :mrgreen:
  • Errare humanum est, perseverare diabolicum... comme déjà dit par un autre intervenant. 

    Tout Shtam est résumé dans cette formule. Quand un découvreur se trompe, et qu'en plus, il refuse de voir l'évidence, alors il se fait allumer. 

    Quand il se trompe et qu'il reconnaît son erreur (beaucoup plus rare), tout va bien. Tout le monde tourne la page, incident immédiatement oublié.

    Et j'ai vu aussi des discussions où le découvreur poste des résultats, et où aucune erreur n'est détectée. Bien sûr, ces discussions génèrent peu de réponses, et donc disparaissent très vite du 'top 20 des discussions récentes'. Mais elles existent aussi.  Et c'est un début de validation. Ca ne prouve pas que la démonstration est correcte, ça prouve qu'elle est suffisamment crédible pour passer un premier niveau de validation.
     
    Tu me dis, j'oublie. Tu m'enseignes, je me souviens. Tu m'impliques, j'apprends. Benjamin Franklin
  • On a un célèbre intervenant qui a dit un truc comme « j’ai envoyé mon travail à des chercheurs et comme je n’ai pas reçu de réponse négative, c’est que c’était un travail juste ». 🤣
    C’est la règle du « qui ne dit mot consent ». 
  • « qui ne dit mot consent »,
    c'est la défense classique du violeur lambda qui devient miraculeusement sourd à l'instant fatidique, ou Alzheimer un peu plus tard.
    Y a des journalistes célèbres de cette race. Y a pas que des journalistes, hélas!
  • Modifié (May 2022)
    Merci Kawkaw d'avoir dévoilé  ta grande trouvaille. Mon compagnon dépasse va devenir riche et j'accepte les miettes.  On ne parle dans les forums que de cette trouvaille. 
    Le 😄 Farceur


  • Je ne sais pas depasse si le parallèle est pertinent. 
  • Dom ton francais est difficile à  déchiffrer 
    Le 😄 Farceur


  • Mince depasse, j'ai loupé le million de dollars :'(Mais finalement je tiens plus à ma dignité qu'au million de dollars :)
  • Modifié (May 2022)
    Math2 te rends-tu compte que toutes ces années,  personne n'a pu remarquer cette idée  rayonnante de Kawkaw 
    Il existe  des entiers positifs  x, y, z  /  z= RC (x^3 + y^3 )  
    Le 😄 Farceur


  • Pauvre Fermat, pauvre Euler, ils n'avaient pas vu ça ! Ils croyaient le contraire ....
  • Modifié (May 2022)
    Bonjour
    Je le savais, je le savais, pour comprendre le pourquoi du comment, nous serons d'accord pour dire que :
    $(a+b)^n=a^{n-1}n_a+b^{n-1}n_b$
    Ensuite si  je considère  un triple pythagoricien, je peux dire
    $a^2=x^2+y^2$
    $a^2=x^2+2bx+b^2$ -> $y^2=2bx+b^2$
    $a^2=c^2+2cy+y^2$ -> $x^2=c^2+2cy$
    $a^2=c^2+2(cy+bx)+b^2$
    Au passage l'on note une propriété (voir mon message en géométrie),
    et l'on retrouve bien la propriété du début de ce message,
    ensuite pour $n>2$ mème cuisine, ici je fais l’hypothèse qu'il existe une solution.
    $a^3=x^3+y^3$
    $a^3=x^3+3x^2b+3xb^2+b^3$
    $a^3=c^3+3c^2y+3cy^2+y^3$
    $a^3=c^3+3c^2y+3cy^2+3x^2b+3xb^2+b^3$
    et maintenant je remarque que cette relation ne peut pas s’écrire sous cette forme $(a+b)^n=a^{n-1}n_a+b^{n-1}n_b$, à cause de
    $3cy^2,3x^2b$ et donc $a^3\ne x^3+y^3$.
    Un avis mathématique ? Parce que pour l'instant et entre nous l'on ne peut pas dire que vous avez été  très  constructifs.
    Cordialement remy
  • Modifié (May 2022)
    Aumeunier a dit :
    nous serons d'accord pour dire que : $(a+b)^n=a^{n-1}n_a+b^{n-1}n_b$
    Explique ?
    Le 😄 Farceur


  • Modifié (May 2022)
    "Un avis mathématique ? parque Parce que pour l'instant et entre nous l'on ne peut pas dire que vous avez été très constructifs.
    Simplement parce que tu ne l'es pas ! Tes mathématiques sont aussi fautives que ton orthographe et ta grammaire !
    "Nous serons d'accord pour dire que : $(a+b)^n = a^{n-1}n_a+b^{n-1}n_b$" (J'ai réduit l'écriture, pas besoin de 3 lignes, sauf si on a des tocs).
    Ben non, à priori, tant qu'on ne nous présente pas $n_a$ et $n_b$ (*) ça n'a pas de sens.
    Je continue :
    "$a^2=x^2+y^2$
    $a^2=x^2+2bx+b^2$ ->$y^2=2bx+b^2$"
    C'est qui $b$ ? D'où sort-il ? Et ce symbole ->? Signifie-t-il $\Rightarrow$ ? Mais pourquoi cette implication ? Puisque tu n'as pas défini $b$, tout ça n'a pas de sens. On ne sait pas ce que tu fais.
    Inutile de continuer, mais prenons la fin : "... cette relation ne peut pas s’écrire sous cette forme $(a+b)^n=a^{n-1}n_a+b^{n-1}n_b$ ..." est
    1) une affirmation non justifiée ("ne pas être écrit" ne veut pas dire "ne pas pouvoir être écrit")
    2) sans rapport avec la suite...
    En bilan : tu ne présentes pas un calcul ou une idée de calcul (il y a peut-être quelque chose dans ton cerveau, mais on n'y est pas), tu écris pour toi, pas pour communiquer. Depuis deux ans on te dit qu'il faut expliquer de quoi tu parles et que ce que tu rédiges doit être compréhensible par les autres (*), que tu ne peux pas être pris au sérieux en continuant ainsi.
    Voilà, j'ai essayé encore une fois de te mener vers une activité sérieuse sur ce forum. À toi de choisir si tu veux être pris au sérieux ou continuer dans les fantaisies.
    Cordialement.
    (*) donc un effort élémentaire de rédaction est utile, ça permet d'être lu plus facilement.
  • Modifié (May 2022)
    Bien vu  merci, mais cela ne change rien au raisonnement, je dois pouvoir  imposer une contrainte pour les puissances impaires et une autre pour les  paires. Une idée de formulation éventuellement ?
    cdl remy.
  • Modifié (May 2022)
    Donc l'idée, à partir d'un  triple pythagoricien
    $z^2= (a+x)^2=x^2+y^2=x^2+2ax+a^2$
    $z^2=(b+y)^2=x^2+y^2=b^2+2by+y^2$
    $z^2= x^2+y^2=b^2+2by+2ax+a^2$
    Je fais la même chose pour $n>2$ dans $(e+f)^n$ et je ne peux pas faire les mêmes mises en facteur, dans les 2 écritures en toute logique, es-tu à la démonstration pour $n=3$, voilà pour l’idée.
    cdl remy.
  • Modifié (May 2022)
    Comment pourrais-je avoir une idée de formulation de ce qui est dans ta tête ? Finalement, tu as une idée, tu ne sais pas quoi en faire, tu es incapable de l'exposer, mais tu demandes aux autres de le faire. C'est d'une ingénuité rare !
    Tout ça pour une situation bien connue depuis plus de 100 ans, qui pour n petit a des preuves élémentaires (Euler pour 3 et 4), pour la plupart des exposants entiers est traitée, et le cas général est démontré depuis presque 30 ans. Du travail comme tu le fais, des centaines de milliers d'amateurs bien plus doués que toi ont présenté des idées sans utilité (dont sans doute la tienne quelle qu'elle soit, tu n'es pas un génie caché, seulement prétentieux, vu ce que tu produis).
    Et si tu commençais par apprendre à écrire les maths correctement ? Pour l'instant, tu te comportes comme un candidat au marathon qui refuse d'apprendre à courir. Être amateur n'oblige pas à être mauvais.
  • Modifié (May 2022)
    Je me suis juste planté sur $(a+b)^n=a^{n-1}n_a+b^{n-1}n_b$ et encore cette relation est vraie pour tout n impair. mais je ne te demande pas de le reconnaître, et je suis d'accord il manque le cas n et pair.
  • Bonjour 
    (citation Aumeunier) Bien vu  merci, mais cela ne change rien au raisonnement  

    Ben non justement !

    gerard0 , vient de te l'expliquer et tu réponds par une autre illusion .... sans explication ni exemple !

    un triplet pythagoricien paramétré par un couple d'entiers $(u , v)$ positif premier entre eux de parité différente  par exemple : $u =2$ et $v =1$

    tel que :
    $u^2 + v^2 = Z= 5$
    $u^2 - v^2 = X= 3$
    $2uv = Y= 4$

    Donc ton $Z² = X^2 + Y^2 = 25$ 

    1) il t'a posé la question : c'est quoi la valeur de $n_a$ et valeur de $n_b$
     
    2) tu indiques une autre version, alors même réponse on connait donc $Z^2 = 25$ et $X^2 = 9$
    d'où : ton  $2ax+a^2 = Y^2 = 16$ et on sait que $X=3$

    Donc on en déduit que $a = 2$ , ok ;
    Alors question : est - ce - que cela serra toujours le cas et pourquoi ou : quelle paramètre te permet d'obtenir directement ton $a$ ?  En conclusion à quoi cela te sert :smile:

    Pour croire que tu peux résoudre de façon générale le cas des puissances $n$ premières ?
    Tu ne pourrais même pas l'utiliser pour démontrer le cas des puissances $2n$ !

    Bref , je continue

    3) J'en déduit que  $b =1$ car $Y = 4$ et ta deuxième équation est vraie , ainsi que la troisième ... et alors ?

    Et en toute logique tu estimes que cela démontre le cas  $n = 3$ par ce que tu ne trouves pas de valeurs  :smile:

    Ou est ta démonstration et donc pourquoi il est impossible de trouver un triplet de cubes ?? parce que tu ne trouves pas de valeurs pour écrire ta formule ?
     
    Ça fait plus de deux siècles qu'on le sait , que l'on ne trouve pas trois cubes dans un triplet pythagoricien et en plus cela a été démontré élémentairement pour prouver que dans les puissances $2n$ avec $n > 2$  un nombre premier, il n'y a pas de solution !

    Regarde et comprend bien la descente infinie de Fermat pour le cas $n = 4$ puis utilises son exemple pour le cas $n = 6$ avec ta logique  .... en te servant du couple de paramètres $(u , v)$ tu verras peut être pourquoi ...!
     
  • Modifié (May 2022)
    Maintenant sommes-nous d’accords pour dire :
    $(a+b)^n=a^{n-1}n_a+b^{n-1}n_b$   pour n =impair, et pour n =pair $(a+b)^n=a^{n-2}n_a+b^{n-2}n_b$.
    Attention un oui même petit, implique le fait que je viens de démontrer le dernier théorème de Fermat.
    Merci pour tout retour, même partiel.
    cdl remy
    ps :  je ne ferai pas de pdf, cette idée me plait bien.
  • J'essaie de résumer tout ça (les propos d'Aumeunier) : 
    Aumeunier dit : Il y a telle propriété, qui est connue depuis quelques siècles, et elle dit qu'à part 1 et 2, aucun entier n ne vérifie telle propriété....
    Le public dit : oui, c'est une conjecture assez connue.
    Aumeunier dit : je sais comment la démontrer.
    Le public dit : vas-y explique.
    Aumeunier dit : Je sais comment faire, il suffit de démontrer que cette propriété est vraie pour tout entier pair (autre que 2), et pour tout entier impair autre que 1.
    Le public dit : exact.
    Aumeunier dit : Alors, pour tout entier pair autre que 2, comment pourrait-on faire ? et pour les impairs, comment pourrait-on faire ?
    Le public dit : ok, tu avais dit que tu avais une solution, mais en fait tu demandes une solution.

    Fin de l'histoire.
    Tu me dis, j'oublie. Tu m'enseignes, je me souviens. Tu m'impliques, j'apprends. Benjamin Franklin
  • LEGLEG
    Modifié (May 2022)
    Au meunier tu dors ? 
    [Maintenant sommes-nous d’accords pour dire :] que tu n'as rien trouvé ...  ça c'est évident, tu ne répètes que tes illusions  .

    Malheureusement : illusion$\neq$démonstration ! Ta formule pour le cas $n=2$ fonctionne parce que l'on a prouvé, démontré, que l'on peut la paramétrer pour tout triplet pythagoricien primitif, et donc tu peux trouver tes $a$ et tes $b$ "sans explication" parce qu'il existe une infinité de solutions 

    Or comme tu ne peux pas prouver qu'il est impossible de paramétrer un triplet de cubes car il y aurait des contradictions, que tu es incapable de définir ... !

    Dis toi bien que ta "" formule "" inutile, ce n'est pas ça, qui permet de prouver que l'on peut paramétrer un triplet pythagoricien primitif quel qu'il soit !

    Tu crois parce que ton $Z - X = a$ et que $Z - Y= b$ que ça prouve le paramétrage d'un triplet pythagoricien, tu n'as même pas été foutu de l'expliquer, ce que je comprends, puisque cela ne sert à rien à part te faire mousser dan tes illusions...
    Bon rêve et ne te réveille pas, car ton moulin va trop vite ... ça fait courant d'air .!
  • Modifié (May 2022)
    Aumeunier, je vais redire ce que disent mes premiers camarades.
    Prenons ta première relation :
    * elle n'est pas quantifiée (il manque des quel que soit, il existe, et les ensembles dans lesquelles les choses vivent). Sans précision ça ne veut rien dire.
    * à supposer que $a$, $b$, $n$ soient donnés (par exemple entiers strictement positifs pour préciser), je veux bien qu'il existe un $n_a$ et un $n_b$ de sorte que ta relation soit vraie. D'ailleurs tu fixes $n_a$ arbitraire dans $\R$, tu auras un $n_b$ et un seul satisfaisant cette relation. Donc je suis d'accord, mais vu que ce n'est pas quantifié, ça m'étonnerait que tu aies voulu dire une telle trivialité sur la résolution des équations du premier degré.
    * j'imagine qu'il faut en plus comprendre $n_a$ et $n_b$ entiers. Alors à ce moment là, c'est déjà un poil moins trivial. Peux-tu donner $n_a$ et $n_b$ lorsque $n=5$ par exemple ?

    Là suite est du même tonneau, et effectivement la cohérence entre les lignes est difficile à déchiffrer.
  • Modifié (May 2022)
    Bonjour
    nous serons d'accord pour dire que :
    $(a+b)^n=a^{n-1}n_a+b^{n-1}n_b$   pour $n$ impair, et pour $n$ pair $(a+b)^n=a^{n-2}n_a+b^{n-2}n_b$.
    Pour rappel $(a+b)^4=a^2(a^2+4ab+3b^2)+b^2(b^2+4ab+3a^2) $

    Ensuite pour $n=3$ ici je fais l’hypothèse qu'il existe une solution.
    $z^3= (a+x)^3=x^3+y^3$
    $z^3=(b+y)^3=b^3+3b^2y+3by^2+y^3$
    $z^3=(a+x)^3=a^3+3a^2x+3ax^2+x^3$
    $z^3= x^3+y^3=(b^3+3b^2y+3by^2)+ (3a^2x+3ax^2+a^3) $
    et maintenant je remarque que cette relation ne peut pas s’écrire sous cette forme $(a+b)^n=a^{n-1}n_a+b^{n-1}n_b$, à cause de
    $3by^2,3ax^2$ et donc $z^3\ne x^3+y^3$.

    Pour $n=4$ ici je fais l’hypothèse qu'il existe une solution.
    $z^4=x^4+y^4$
    $z^4= (b+x)^4=x^4+4x^3b+6x^2b^2+4xb^3+b^4$
    $z^4= (a+y)^4=y^4+4y^3a+6y^2a^2+4ya^3+a^4$
    $z^4=(4y^3a+6y^2a^2+4ya^3+a^4 )+(4x^3b+6x^2b^2+4xb^3+b^4)$
    et maintenant je remarque que cette relation ne peut pas s’écrire sous cette forme $(a+b)^n=a^{n-2}n_a+b^{n-2}n_b$, à cause de $4x^3b,4y^3a$ et donc $z^4\ne x^4+y^4$

    Pour tout $n$ j'aurai obligatoirement un reste qui ne pourra pas être mis en facteurs.
    cdl remy
  • Modifié (May 2022)
    Maintenant la version je suis complétement bouché.
    $z^n= (a+x)^n=x^n+y^n$
    Dans cette relation $(a+x)^n$ j’impose $x^n$ dans le développement $(a+x)^n$
    ce qui veut dire que tout le reste et égal à $y^n$ ben oui  $z= (\ldots)+x^n$ je fais la même chose avec $y$ j’impose $y^n$ dans le développement $(b+y)^n$ce qui veut dire que tout le reste et égal à $x^n$ ben oui  $z= (\ldots)+y^n$ Donc $ (\ldots)+ (\ldots)=z^n$ puis je remarque que cette représentation ne peut pas être mise en facteurs comme une représentation normale de $(e+f)^n=z^n$.
    Voilà cela devrais être plus simple.
    cdl remy.

  • À partir d'une relation vraie pour n = 3, vous en déduisez une relation pour tout n impair.
    À partir d'une relation vraie pour n = 4, vous en déduisez une relation pour tout n pair.

    C'est osé de faire une induction à partir d'un seul cas, et, vous allez rire, c'est complètement faux.

    Il ne faut pas respirer la compote, ça fait tousser.

    J'affirme péremptoirement que toute affirmation péremptoire est fausse
  • Modifié (May 2022)
    Oui la première affirmation (avec les quantificateurs que l'on imagine) est grossièrement fausse à partir de $n=5$.
  • Modifié (May 2022)
    non pas vraiment parque la somme des puissances et une constante,la chose importante ce n'est pas les coefs multiplicateurs  mais la possibilité de mètre en facteur une puissance de a et de b
    Dans les pires des cas s'il n'y a rien à ce sujet dans vos tablettes, je peux ne pas utiliser cette relation et me contenter de mètres en évidence le fait que ces deux expressions ne peuvent pas avoir la même mise en facteurs pour une puissance de n donner, mais cela ces pas évidents à expliquer.
     cdl remy

  • « mètres en évidence »
    waaw
Cette discussion a été fermée.
Success message!